matheraum.de
Raum für Mathematik
Offene Informations- und Nachhilfegemeinschaft

Für Schüler, Studenten, Lehrer, Mathematik-Interessierte.
Hallo Gast!einloggen | registrieren ]
Startseite · Forum · Wissen · Kurse · Mitglieder · Team · Impressum
Forenbaum
^ Forenbaum
Status Schulmathe
  Status Primarstufe
  Status Mathe Klassen 5-7
  Status Mathe Klassen 8-10
  Status Oberstufenmathe
    Status Schul-Analysis
    Status Lin. Algebra/Vektor
    Status Stochastik
    Status Abivorbereitung
  Status Mathe-Wettbewerbe
    Status Bundeswettb. Mathe
    Status Deutsche MO
    Status Internationale MO
    Status MO andere Länder
    Status Känguru
  Status Sonstiges

Gezeigt werden alle Foren bis zur Tiefe 2

Navigation
 Startseite...
 Neuerdings beta neu
 Forum...
 vorwissen...
 vorkurse...
 Werkzeuge...
 Nachhilfevermittlung beta...
 Online-Spiele beta
 Suchen
 Verein...
 Impressum
Das Projekt
Server und Internetanbindung werden durch Spenden finanziert.
Organisiert wird das Projekt von unserem Koordinatorenteam.
Hunderte Mitglieder helfen ehrenamtlich in unseren moderierten Foren.
Anbieter der Seite ist der gemeinnützige Verein "Vorhilfe.de e.V.".
Partnerseiten
Weitere Fächer:

Open Source FunktionenplotterFunkyPlot: Kostenloser und quelloffener Funktionenplotter für Linux und andere Betriebssysteme
StartseiteMatheForenMathematik-WettbewerbeNeue Aufgabe Nr. 5
Foren für weitere Schulfächer findest Du auf www.vorhilfe.de z.B. Geschichte • Erdkunde • Sozialwissenschaften • Politik/Wirtschaft
Forum "Mathematik-Wettbewerbe" - Neue Aufgabe Nr. 5
Neue Aufgabe Nr. 5 < Wettbewerbe < Schule < Mathe < Vorhilfe
Ansicht: [ geschachtelt ] | ^ Forum "Mathematik-Wettbewerbe"  | ^^ Alle Foren  | ^ Forenbaum  | Materialien

Neue Aufgabe Nr. 5: Übungsaufgabe
Status: (Übungsaufgabe) Übungsaufgabe Status 
Datum: 17:59 Do 17.02.2005
Autor: Hanno

Hallo an alle!

Sei $S$ die Menge aller ungeraden, natürlichen Zahlen größer 1. Sei ferner [mm] $\delta [/mm] (x)$ für jede Zahl [mm] $x\in [/mm] S$ diejenige eindeutige ganze Zahl, für die [mm] $2^{\delta (x)} Für [mm] $a,b\in [/mm] S$ sei weiter [mm] $a\* b:=2^{\delta(a)-1}(b-3)+a$. [/mm]

Man zeige, dass
(i) [mm] $\*$ [/mm] auf $S$ abgeschlossen ist
(ii) [mm] $\*$ [/mm] assoziativ ist


Liebe Grüße,
Hanno

        
Bezug
Neue Aufgabe Nr. 5: Tip
Status: (Mitteilung) Reaktion unnötig Status 
Datum: 15:26 Fr 18.02.2005
Autor: Hanno

Hallo an alle!

Für jede Zahl [mm] $a\in \IN$ [/mm] gibt es Koeffizienten [mm] $a_0,a_1,...,a_{n_0}\in \{0,1\}$ [/mm] so, dass [mm] $a=\summe_{i=0}^{n_0}{a_i 2^i}$ [/mm] gilt.

Liebe Grüße,
Hanno

Bezug
        
Bezug
Neue Aufgabe Nr. 5: (i)
Status: (Frage) beantwortet Status 
Datum: 23:14 Fr 18.02.2005
Autor: Max

Hi,

ich denke (i) schaffe ich noch *hoff*

> Man zeige, dass
>  (i) [mm]\*[/mm] auf [mm]S[/mm] abgeschlossen ist
>  (ii) [mm]\*[/mm] assoziativ ist

Die Verknüpfung [mm] $\*$ [/mm] ist auf $S$ abgeschlossen, wenn für alle $a,b [mm] \in [/mm] S$ gilt [mm] $a\* [/mm] b [mm] \in [/mm] S$.

Da [mm] $2^{\delta(a)-1}(b-3)>0$ [/mm] und gerade, ist die Summe [mm] $a\* b=2^{\delta(a)-1}(b-3)+a$ [/mm] ungerade. Da [mm] $a\in [/mm] S$ ist $a>1$, damit ist auch [mm] $2^{\delta(a)-1}(b-3)+a [/mm] >1$. Also ist [mm] $a\* [/mm] b [mm] \in [/mm] S$.

Entweder ich bin doof oder Hanno hat sich verschrieben....

Es gilt $ [mm] 2^2 [/mm] < 5 < [mm] 2^3 [/mm] $, also [mm] $\delta(5)=2$. [/mm] Es gilt [mm] $2^3<11<2^4$, [/mm] also [mm] $\delta(11)=3$. [/mm]

[mm] $5\* [/mm] 11 = [mm] 2^{2-1}(11-3)+5 [/mm] = 2 [mm] \cdot [/mm] (11-3)+5 [mm] =2\cdot [/mm] 8+5 =21$

[mm] $11\* [/mm] 5 = [mm] 2^{3-1}(5-3)+11= 2^2\cdot [/mm] (5-3)+11= [mm] 4\cdot [/mm] 2 +11 =19$

Das wäre damit ein Gegenbeispiel, oder habe ich mich irgendwo geirrt?

Gruß Brackhaus

Bezug
                
Bezug
Neue Aufgabe Nr. 5: kommutativ vs. assoziativ
Status: (Mitteilung) Reaktion unnötig Status 
Datum: 10:02 Sa 19.02.2005
Autor: Plantronics

Morgen,

meiner Meinung prüfts du die kommutativität nach a*b=b*a und nicht die verlangte
Assoziativität (a*b)*c=a*(b*c)=a*b*c, und ohne Beweis würde ich sagen
1., es ist nicht kommutativ
und 2., es ist assoziativ (rein dem Gefühl nach)

mfg,
Martin

Bezug
                        
Bezug
Neue Aufgabe Nr. 5: Mitteilung
Status: (Mitteilung) Reaktion unnötig Status 
Datum: 11:48 Sa 19.02.2005
Autor: Max

Uuuups, ja das ist natürlich peinlich. Also trifft "oder ich bin blöd" zu ;-)

Bezug
                
Bezug
Neue Aufgabe Nr. 5: Antwort
Status: (Antwort) fertig Status 
Datum: 15:27 Sa 19.02.2005
Autor: Hanno

Hallo Max!

Deine Lösung zu (i) ist völlig korrekt [ok]! Der Irrtum, der dir im Folgenden unterlaufen ist, ist ja bereits geklärt. Nun also ran an Aufgabe (ii)!

Liebe Grüße,
Hanno

Bezug
        
Bezug
Neue Aufgabe Nr. 5: Lösung
Status: (Mitteilung) Reaktion unnötig Status 
Datum: 08:36 Mi 23.02.2005
Autor: Hanno

Eingabefehler: "{" und "}" müssen immer paarweise auftreten, es wurde aber ein Teil ohne Entsprechung gefunden (siehe rote Markierung)

Hallo an alle!

Ich poste einfach mal die Lösung, da an dieser Aufgabe niemand mehr zu arbeiten scheint:

Seien $a,b,c\in S$ mit $a:=\summe_{i=0}^{n_{0}(a)}{a_i 2^i}, b:=\summe_{i=0}^{n_{0}(b)}{b_i 2^i}, c:=\summe_{i=0}^{n_{0}(c)}{c_i 2^i}\quad a_i,b_i,c_i\in \{0,1\}$. Da $a,b,c$ ungerade sind, sind die $a_0,b_0,c_0=1$. Somit gilt $\delta (a)=n_{0}(a), \delta (b)=n_{0}(b), \delta (c)=n_{0}(c)$ und daher auch $a\* b:=2^{\delta(a)-1}(b-3)+a=2^{n_0(a)-1}(b-3)+a$. Existiert ein $i, 1\leq i<n_0(b)$ mit $b_i=1$, dann ist $\delta(b-3)=\delta(b)=n_0(b)$ und $\delta(2^{n_0(a)-1}(b-3))=n_0(a)+n_0(b)-1$. Zur besseren Lesbarkeit sei $b':=2^{n_0(a)-1}(b-3)$. Dann muss $\delta(b')=\delta(b'+a)$ gelten, da nach Voraussetzung die Menge der $b_i, 1\leq i<n_0(b)$ mit $b_i=1$ nichtleer ist; sie besitzt ein kleinstes Element $i_0$. Durch Subtraktion von $3$ wird dieser Koeffizient auf $0$ gesetzt, folglich ist $b'_{n_0(a)-1+i_0}=0$. Wird nun $a$ addiert, so kann nur dann $\delta(b')\not= \delta(b'+a)$ gelten, wenn für alle $i, n_0(a)\leq n_0(b')$ der Koeffizient $b'_i$ den Wert 1 hat. Dies widerspricht allerdings $b'_{n_0(a)-1+i_0}=0$. Somit ist tatsächlich $\delta(b'+a)=\delta(b')$.
Eingesetzt in $(a\* b)\* c$ ergibt dies $(a\* b)\* c=2^{n_0(a)-1+n_0(b)-1}(c-3)+2^{n_0(a)-1}(b-3)+a=2^{n_0(a)-1}((2^{n_0(b)-1}(c-3)+b)-3)+a=a\* (b\* c)$.
Ist hingegen $b_i=0$ für alle $1\leq i<n_0(b)$, so ist $\delta(2^{n_0(a)-1}(b-3)}=n_0(a)+n_0(b)-2$. Allerdings ist genau dann auch $\delta(b'+a)=\delta(b')+1$, da alle Koeffizienten $b'_{n_0(a)-1+i}, 1\leq i\leq n_0(b)-1$ den Wert 1 haben, durch Addition von $a$ der Koeffizient $n_0(a)$ erhöht wird, alle Koeffizienten $b'_{n_0(a)-1+i}, 1\leq i\leq n_0(b)-1$ den Wert 0 annehmen und daher $b'_{n_0(a)+n_0(b)+1}=1$ gilt. Daher gilt auch für diesen Fall $\delta(2^{n_0(a)-1}(b-3))=n_0(a)+n_0(b)-1$ und die Behauptung folgt abermals aus obiger Rechnung.


Damit ist gezeigt, dass $\*$ assoziativ ist.


Liebe Grüße,
Hanno

Bezug
Ansicht: [ geschachtelt ] | ^ Forum "Mathematik-Wettbewerbe"  | ^^ Alle Foren  | ^ Forenbaum  | Materialien


^ Seitenanfang ^
www.schulmatheforum.de
[ Startseite | Forum | Wissen | Kurse | Mitglieder | Team | Impressum ]